Traffic speed: The mean speed for a sample of cars at a certain intersection was kilometers per hour with a standard deviation of kilometers per hour, and the mean speed for a sample of motorcycles was kilometers per hour with a standard deviation of kilometers per hour. Construct a confidence interval for the difference between the mean speeds of motorcycles and cars at this intersection. Let denote the mean speed of motorcycles and round the answers to at least two decimal places. A confidence interval for the difference between the mean speeds, in kilometers per hour, of motorcycles and cars at this intersection is ________.
Construct the 98% confidence interval for the difference μ 1-y 2 when x 1 475.12, x 2-32134, s 1-43.48, s 2-21.60, n 1-12, and n 2-15. Use tables to find the critical value and round the answers to two decimal places. A 98% confidence interval for the difference in the population means is ________.

Answers

Answer 1

Answer:

Step-by-step explanation:

Hello!

X₁: speed of a motorcycle at a certain intersection.

n₁= 135

X[bar]₁= 33.99 km/h

S₁= 4.02 km/h

X₂: speed of a car at a certain intersection.

n₂= 42 cars

X[bar]₂= 26.56 km/h

S₂= 2.45 km/h

Assuming

X₁~N(μ₁; σ₁²)

X₂~N(μ₂; σ₂²)

and σ₁² = σ₂²

A 90% confidence interval for the difference between the mean speeds, in kilometers per hour, of motorcycles and cars at this intersection is ________.

The parameter of interest is μ₁-μ₂

(X[bar]₁-X[bar]₂)±[tex]t_{n_1+n_2-2}[/tex] * [tex]Sa\sqrt{\frac{1}{n_1} +\frac{1}{n_2} }[/tex]

[tex]t_{n_1+n_2-2;1-\alpha /2}= t_{175; 0.95}= 1.654[/tex]

[tex]Sa= \sqrt{\frac{(n_1-1)S_1^2+(n_2-1)S_2^2}{n_1+n_2-2} } = \sqrt{\frac{134*16.1604+41*6.0025}{135+42-2} } = 3.71[/tex]

[(33.99-26.56) ± 1.654 *([tex]3.71*\sqrt{\frac{1}{135} +\frac{1}{42} }[/tex])]

[6.345; 8.514]= [6.35; 8.51]km/h

Construct the 98% confidence interval for the difference μ₁-μ₂ when X[bar]₁= 475.12, S₁= 43.48, X[bar]₂= 321.34, S₂= 21.60, n₁= 12, n₂= 15

[tex]t_{n_1+n_2-2;1-\alpha /2}= t_{25; 0.99}= 2.485[/tex]

[tex]Sa= \sqrt{\frac{(n_1-1)S_1^2+(n_2-1)S_2^2}{n_1+n_2-2} } = \sqrt{\frac{11*(43.48)^2+14*(21.60)^2}{12+15-2} } = 33.06[/tex]

[(475.12-321.34) ± 2.485 *([tex]33.06*\sqrt{\frac{1}{12} +\frac{1}{15} }[/tex])]

[121.96; 185.60]

I hope this helps!


Related Questions

Explain what the difference between a tangent and a secant segment is?

Answers

Answer:

A tangent line touches a curve at one point and has the same slope as the curve at that point. A secant line intersects at 2 or more points and has a slope equal to the average rate of change between those points.

Answer:

A Tangent of a circle is found outside of the circle but touching 2 points of the circle on the outside. But a Secant is found inside the circle and it touches 2 points in the circle. A Chord can always be Secant, but a secant can not always be a chord because it may pass through the circle.

Step-by-step explanation:

I’ve been stuck can someone explain

Answers

Answer:

Option (3)

Step-by-step explanation:

From the figure attached,

AB and CD are two chords intersecting at O.

m∠AOD = 37°

m∠AOC + m∠AOD = 180° [Since these angles are supplementary angles]

m∠AOC = 180° - 37°

              = 143°

By the theorem of intersecting chords,

Measure of angle formed is the half of the sum of measures of the arcs intercepted by the angle and vertical angle.

m∠AOC = [tex]\frac{1}{2}(\widehat{AC}+\widehat{BD})[/tex]

143° = [tex]\frac{1}{2}[(x+5)+(x-5)][/tex]

143° = x

Therefore, Option (3) will be the answer.

A line has an equation of y = - 3x + 8. What is the y-intercept of the line? Please enter your answer as a coordinate (x, y). *

Answers

Answer:

(0, 8).

Step-by-step explanation:

The y intercept occurs when  = 0, so we have the equation:

y = -3(0) + 8

y = 0 + 8

y = 8.

The answer is (0, 8).

The average cost of producing a single bicycle based on the total number of bicycles produced, x, is represented by this function:f(x)=800+30,000/xUse the drop-down menus to complete the statements to show the difference between the mathematical and reasonable ranges. The reasonable range includes the set of A whole number, B rational numbers,C integers, D real numbers where 800 < y ≤ , A 801, B 30,000, C 30,800, D 24,000,000 while the mathematical range includes the set of A whole numbers,B rational numbers C integers D real numbers and only excludes the value . A 0, B 800, C 30,800

Answers

Answer:

The correct answers are B, C, D, B

Step-by-step explanation:

These answers complete the statements that show the difference between the mathematical and reasonable ranges. Good luck! :)

B) Rational Numbers

C) 30,800

D) Real Numbers

B) 800

Correct on edge2020!

Answer:

BCDB

Step-by-step explanation:

Stan ran 4 7/10 miles , which was 1 3/10 fewer miles than Matt ran. For students wrote and solve the equation to find him the number of miles that Matt Ryan which student wrote and solve the equation correctly

Answers

Answer:

Matt ran 6 miles.

Step-by-step explanation:

Stan ran 4 and 7/10 miles, and this is 1 and 3/10 fewer miles than Matt.

this means that Matt ran the following amount of miles

4 and 7/|0 + 1 and 3/10 miles:

(4 + 1) + (7/10 + 3/10) = 5 + 10/10 = 6 miles.

This would be the correct way to solve this equation.

Hurry please! I really need help For what value of the variable is the value of 3−5c one less than the value of 1−c?

Answers

Answer:

Any values that is greater than 1/2.

Step-by-step explanation:

You have to form an inequality. Given that what values is added to c will satisfy that 3 - 5c is less than 1 - c :

[tex]3 - 5c < 1 - c[/tex]

Next, you have to solve :

[tex] - 5c + c < 1 - 3[/tex]

[tex] - 4c < - 2[/tex]

[tex]c > \frac{ - 2}{ - 4} [/tex]

[tex]c > \frac{1}{2} [/tex]

Mr. Evans is considering offering a second after-school tutoring session for his math students each week. He records the number of
students who attend his current sessions each week. The results from the last twelve weeks are shown in the dot plot below.
+
1 2
+
0
+
3
4 5 6 7 8 9 10 11 12 13 14 15
Number of Students
Which statement properly describes the data?
There is not enough information to determine if the data is skewed.
O
The data is symmetric.
The data is skewed right.
The data is skewed left.

Answers

Answer:

Step-by-step explanation:

the data is skewed left

The data in the given set is skewed towards left because the mean of the data is 7.5 and the data is not distributed symmetrically in the given set. Thus, the correct option is C.

What is a Skewed data set?

Skewed data is the data which creates an asymmetrical, skewed curve on the graph scale. In statistics, the graph of a data set with normal distribution is symmetrical and has a bell-like shape. However, the skewed data has a tail on either side of the graph as well.

Skewness can be demonstrated on a bell curve when the data points are not distributed symmetrically to the left and right sides of the median on the curve. If the bell curve is shifted to the left or towards the right side, it is said to be skewed curve. The two halves of the distribution are not mirror images in the skewed curve because the data are not distributed equally on both sides of the distribution peak.

Therefore, the correct option is C.

Learn more about Skewed data here:

https://brainly.com/question/3907939

#SPJ2

Pls answer anyone out there pls pls pls

Answers

Answer:

See below.

Step-by-step explanation:

9.

Property of a rhombus:

In a rhombus, the diagonals are perpendicular.

The sum of the measures of the angles of a triangle is 180 deg.

Since the diagonals are perpendicular, the angles formed by the intersection of the diagonals are right angles and measure 90 deg.

m<ABD + m<CAB + 90 = 180

50 + m<CAB + 90 = 180

m<CAB + 140 = 180

(i) m<CAB = 40

The diagonals of a rhombus divide the rhombus into 4 congruent triangles.

Call the point of intersection of the diagonals E.

Triangles CEB and AEB are congruent.

m<BCA = m<DCA = 40

m<BCD = m<BCA + m<DCA = 40 + 40

(ii) m<BCD = 80

m<CDB = m<ADB = 50

m<ADC = m<CDB + m<ADB = 50 + 50

(iii) m<ADC = 100

10.

There are two angles labeled z. One is near point E and one is near point O. One of them probably is x.

Two angles measure 60 and 80. Add them to get 140.

z (near point O) and the 140 deg angle are a linear pair. their measures add to 180 deg.

z + 140 = 180

z = 40 (This is the z near point O.)

z (near point O) and 60 deg add to an interior angle of the parallelogram.

z + 60 = 40 + 60 = 100

The interior angle at vertex O measures 100 deg.

Adjacent interior angles of a parallelogram are supplementary.

100 + y = 180

y = 80

The two angles labeled z are alternate interior angles. Since the sides of a parallelogram are parallel, the two angles labeled z are congruent and measure 40 deg.

z = 40 (This is angle z near point E)

Abcd is a trapozium what Is the the value of x of a is 110

Answers

Answer:

70°

Step-by-step explanation:

Since it's a trapezoid the sum of a and x would be 180°

110° + x = 180°

x = 70°

9. A pyramid has a height of 10 inches and a base with an area of 21 square inches.

Find the volume of the pyramid.

F210 in

G 105 in

H 70 in

J 35 in

Answers

Answer:

  H  70 in³

Step-by-step explanation:

The volume of a pyramid is given by the formula ...

  V = (1/3)Bh

  V = (1/3)(21 in²)(10 in) = 70 in³

The volume of the pyramid is 70 cubic inches.

Answer:

[tex]= 70 {in}^{3}[/tex]

Third answer is correct.

Step-by-step explanation:

[tex]v = \frac{base \: \: \: area \times height}{3} \\ = \frac{21 \times 10}{3} \\ = 70 {in}^{3} [/tex]

hope this helps

brainliest appreciated

good luck! have a nice day!

A MP3 Manufacturer claims that 65% of teenagers have their own MP3 player. A researcher wishes to test the claim and selects a random sample of 80 teenagers. She finds that 57 have their MP3 player. At a .05 significance level, should the claim be rejected? Please show work.

Answers

Answer:

Null hypothesis: H0 = 0.65

Alternative hypothesis: Ha ≠ 0.65

z = 1.172

P value = P(Z≠1.172) = 0.24

Decision we fail to reject the null hypothesis. That is, there is convincing evidence enough to reject the Null hypothesis.

Rule

If;

P-value > significance level --- accept Null hypothesis

P-value < significance level --- reject Null hypothesis

Z score > Z(at 95% confidence interval) ---- reject Null hypothesis

Z score < Z(at 95% confidence interval) ------ accept Null hypothesis

Step-by-step explanation:

Given;

n=80 represent the random sample taken

Null hypothesis: H0 = 0.65

Alternative hypothesis: Ha ≠ 0.65

Test statistic z score can be calculated with the formula below;

z = (p^−po)/√{po(1−po)/n}

Where,

z= Test statistics

n = Sample size = 80

po = Null hypothesized value = 0.65

p^ = Observed proportion = 57/80 = 0.7125

Substituting the values we have

z = (0.7125-0.65)/√(0.65(1-0.65)/80)

z = 1.17201807734

z = 1.172

To determine the p value (test statistic) at 0.05 significance level, using a two tailed hypothesis.

P value = P(Z≠1.172) = 0.241197 = 0.24

Since z at 0.05 significance level is between -1.96 and +1.96 and the z score for the test (z = 1.172) which falls within the region bounded by Z at 0.05 significance level. And also the one-tailed hypothesis P-value is 0.24 which is higher than 0.05. Then we can conclude that we have enough evidence to FAIL to reject the null hypothesis, and we can say that at 5% significance level the null hypothesis is valid.

A person must pay $ $ 8 to play a certain game at the casino. Each player has a probability of 0.21 of winning $ $ 14, for a net gain of $ $ 6 (the net gain is the amount won 14 minus the cost of playing 8). Each player has a probability of 0.79 of losing the game, for a net loss of $ $ 8 (the net loss is simply the cost of playing since nothing else is lost). What is the Expected Value for the player (that is, the mean of the probabiltiy distribution)? If the Expected Value is negative, be sure to include the "-" sign with the answer. Express the answer with two decimal places.

Answers

Answer:

-$5.06

Step-by-step explanation:

Given the probability distribution of X where X is the net gain or loss

[tex]\left|\begin{array}{c|c|c}Profit(X)&\$6&-\$8\\P(X)&0.21&0.79\end{array}\right|[/tex]

The expected value of X is defined as follows:

Expected Value of X, [tex]E(X)=\sum_{i=1}^nx_iP(x_i)[/tex]

Therefore, the expected value of the player

E(X)=(6*0.21)+(-8*0.79)

=1.26-6.32

[tex]E(X)=-\$5.06[/tex]

The expected value of each player at the casino is -$5.06.

Pleaseeee Thank you!!!!!!!!!

Answers

Answer:

D.

Step-by-step explanation:

Let's solve each choice.

A: 3 divided by 90 is 3/90 and can be simplified to 1/30. So no.

B: 1/5 divided by 6. Recall that when dividing, you multiply the 1st term by the 2nd term's reciprocal. If you do this, you get 1/30. So no.

C: 1/6 divided by 5. Again, multiply 5's reciprocal to 1/6. There you get 1/30. So no.

D: 6 divided by 1/5. Again, multiply 1/5's reciprocal to 6. 1/5's reciprocal is 5. 5*6 = 30. So this is the correct answer.

The table shows the number of heartbeats in minutes for Shen and Adrian.

A 3-column table with 2 rows. Column 1 is labeled Person with entries Shen, Adrian. Column 2 is labeled Heartbeats with entries 192, 360, Column 3 is labeled Time (minutes) with entries 3, 5.

Which statements are true? Check all that apply.
Shen has a slower heart rate than Adrian.
Adrian has a slower heart rate than Shen.
Shen’s unit heart rate is 64 beats per minute.
Adrian’s unit heart rate is 72 beats per minute.
Adrian’s unit heart rate is 120 beats per minute.

Answers

Answer:

Shen has a slower heart rate than Adrian.

Shen’s unit heart rate is 64 beats per minute.

Adrian’s unit heart rate is 72 beats per minute.

An epidemiologist found five cases of "big toe cancer" in the Yukon Territory. Because there were only a few cases, the epidemiologist decided to conduct a matched case-control study to determine whether shoe size larger than 9 is a risk factor for big toe cancer. Cases were individually matched to one control for daily activity, history of athlete’s foot, and history of ingrown toenails. The following data were gathered:
Shoe size > 9
Pair Case Control
1 Yes No
2 No No
3 No Yes
4 Yes Yes
5 No Yes
Compute the proper measure of association.
Interpret your results.
If you were to investigate a rare cancer in Lynchburg, where might you look for data?
What would be necessary legally and ethically to be able to utilize this data set(s)?
Submit your thread by 11:59 p.m. (ET) on Thursday of Module/Week 3, and submit your replies by 11:59 p.m. (ET) on Sunday of the same module/week.

Answers

Answer:

Step-by-step explanation:

Given that:

An epidemiologist found five cases of "big toe cancer" in the Yukon Territory.

Therefore, shoe size > 9

1) From the required data given below

                       Case       Control       Total

Yes                2(a)             3(b)            5

No                  3(c)             2(d)            5

Total               5                 5                10

∴ odds ratio = ad/bc

= 4/9

=0.444

2) From the less than 1.0 mean that the odds of cancer among case is lower than the odds of cancer among controls

Choose the correct answer below. A. The mean MPG of this type of vehicle for 95​% of all samples of the same size is contained in the interval. B. 95​% of the sample data fall between the limits of the confidence interval. C. We have 95​% confidence that the mean MPG of this type of vehicle for the sample is contained in the interval. D. We have 95​% confidence that the population mean MPG of this type of vehicle is contained in the interval.

Answers

Answer:

Step-by-step explanation:

The question is incomplete. The complete question is

The table below contains the overall miles per gallon (MPG) of a type of vehicle. Complete parts a and b below.

28, 34, 28, 20, 21, 31, 28, 24, 34, 35 , 36, 26, 25, 20

a. Construct a 95% confidence interval estimate for the population mean MPG for this type of vehicle, assuming a normal distribution.

b. Choose the correct answer below.

A. We have 95% confidence that the mean MPG of this type of vehicle for the sample is contained in the interval.

B. We have 95 ℅ confidence that the population mean MPG of this type of vehicle is contained in the interval. This is the correct answer.

C.95 % of the sample data fall between the limits of the confidence interval.Your answer is not correct.

D. The mean MPG of this type of vehicle for 95?% of all samples of the same size is contained in the interval.

Solution:

a) Mean = (28 + 34 + 28 + 20 + 21 + 31 + 28 + 24 + 34 + 35 + 36 + 26 + 25 + 20)/14 = 27.86

Standard deviation = √(summation(x - mean)²/n

n = 14

Summation(x - mean)² = (28 - 27.86)^2 + (34 - 27.86)^2 + (28 - 27.86)^2 + (20 - 27.86)^2 + (21 - 27.86)^2+ (31 - 27.86)^2 + (28 - 27.86)^2 + (24 - 27.86)^2 + (34 - 27.86)^2 + (35 - 27.86)^2 + (36 - 27.86)^2 + (26 - 27.86)^2 + (25 - 27.86)^2 + (20 - 27.86)^2 = 399.7144

Standard deviation = √(399.7144/14) = 5.34

Confidence interval is written in the form,

(Sample mean - margin of error, sample mean + margin of error)

The sample mean, x is the point estimate for the population mean.

Margin of error = z × s/√n

Where

s = sample standard deviation = 5.34

n = number of samples = 14

From the information given, the population standard deviation is unknown and the sample size is small, hence, we would use the t distribution to find the z score

In order to use the t distribution, we would determine the degree of freedom, df for the sample.

df = n - 1 = 14 - 1 = 13

Since confidence level = 95% = 0.95, α = 1 - CL = 1 – 0.95 = 0.05

α/2 = 0.05/2 = 0.025

the area to the right of z0.025 is 0.025 and the area to the left of z0.025 is 1 - 0.025 = 0.975

Looking at the t distribution table,

z = 2.16

Margin of error = 2.16 × 5.34/√14

= 3.08

The confidence interval is 27.86 ± 3.08

b) B. We have 95 ℅ confidence that the population mean MPG of this type of vehicle is contained in the interval.

a rectangle has an area of 54 square inches and a length of 6 inches. what is the width, in inches, of the rectangle?​

Answers

Answer:

9 inches

Step-by-step explanation:

For similar problems like this, divide the area by the given length or width. In this case, your equation would be 54/6 = 9.

The width of the rectangle can be found as 9 inch.

How to solve a linear equation?

A linear equation can be solved by equating the LHS and RHS of the equation following some basic rules such as by adding or subtracting the same numbers on both sides and similarly, doing division and multiplication with the same numbers.

The area and length of rectangle are given as 54 inch² and 6 inches.

Suppose the width of the rectangle be x.

Since, the area of rectangle is given as the product of length and breadth, the following equation can be written as,

6x = 54

⇒ x = 54/6

⇒ x = 9

Hence, the width is given as 9 inch.

To know more about linear equation click on,

https://brainly.com/question/11897796

#SPJ2

PLEASE HELP QUICKLY AS POSSIBLE THANK YOU :)​

Answers

Answer:

Rhombus

Step-by-step explanation:

Answer:

Step-by-step explanation:

rhombus

Given the speeds of each runner below, determine who runs the fastest. \text{Noah runs 11 feet per second.} Noah runs 11 feet per second. \text{Katie runs 423 feet in 33 seconds.} Katie runs 423 feet in 33 seconds. \text{Jake runs 1 mile in 396 seconds.} Jake runs 1 mile in 396 seconds. \text{Liz runs 638 feet in 1 minute.} Liz runs 638 feet in 1 minute.

Answers

Answer:

Jake

Step-by-step explanation:

Noah: 11 feet per second

Katie: 423 feet / 33 seconds = 12.82 ft/sec (just divide the feet / seconds)

Jake:

1 mile = 5280 feet

Adam runs 5280ft / 396 seconds = 13.34 ft/sec

Liz:

1 minute = 60 second.

Liz runs 638 feet / 60 seconds = 10.63 ft / sec

From the above results we find that Jake runs the fastest

Drag the tiles to the correct boxes to complete the pairs. Not all tiles will be used
The box plot compares the monthly average temperature (in degrees Fahrenheit) recorded in the towns of Springwood and Meadows from April
to October Match each phrase to its correct value.
Meadows
Springwood
60
65
70
75
30
85
00
95
91
86
80
73
14
12
6
1
2
the median of the temperatures at Springwood
the median of the temperatures at Meadows
the interquartile range of the temperatures at Springwood
the interquartile range of the temperatures at Meadows
(ANSWER ASAP FOR 30 POINTS.)

Answers

12 your welcome if I help you

Answer:

12

Step-by-step explanation:

What’s the correct answer for this?

Answers

Answer:

36

Step-by-step explanation:

In circle with center O,

[tex] chord\overline {EF} \cong chord\overline {WV}... (Given) [/tex]

Since, congruent chords are equidistant from the center of the circle.

[tex] \therefore PG = GO\\

\therefore - x +10 = - 3(x+2)\\

\therefore - x + 10 = - 3x - 6\\

\therefore 3x - x = - 6-10\\

\therefore 2x = - 16\\\\

\therefore x = \frac{-16}{2} \\\\

\huge \red {\boxed {\therefore x = - 8}} \\\\

\because \overline {PO} = \overline {PG} + \overline {GO} \\

\therefore \overline {PO} = - x + 10 + \{-3(x + 2)\}\\

\therefore \overline {PO} = - x + 10 - 3x - 6\\

\therefore \overline {PO} = - 4x + 4 \\

\therefore \overline {PO} = - 4\times (-8)+ 4 \\

\therefore \overline {PO} =32+ 4 \\

\huge \orange {\boxed {\therefore \overline {PO} =36}} \\[/tex]

What is the measure of arc WXY

Answers

Answer:

152°

Step-by-step explanation:

Let P be any point on tangent [tex] \overleftrightarrow{YZ} [/tex] and WY is secant or chord of the [tex] \odot J[/tex] .

[tex] \therefore m\angle WYZ + m\angle WYP = 180°\\(Straight \: line \: \angle 's) \\

\therefore 104° + m\angle WYP = 180°\\

\therefore m\angle WYP = 180°- 104° \\

\red{\boxed {\bold {\therefore m\angle WYP = 76°}}} \\[/tex]

NOW, by tangent secant theorem:

[tex] m\angle WYP =\frac{1}{2}\times m(\widehat{WXY}) \\\\

76°=\frac{1}{2}\times m( \widehat{WXY}) \\\\

76°\times 2 =m( \widehat{WXY}) \\

\huge \purple {\boxed {\therefore m(\widehat{WXY}) = 152°}} [/tex]

What is the simplified form of this expression?

Answers

Answer:

[tex]=13x+5[/tex]

Step-by-step explanation:

[tex]\left(2x+9\right)+\left(11x-4\right)\\\mathrm{Remove\:parentheses}:\quad \left(a\right)=a\\=2x+9+11x-4\\\mathrm{Group\:like\:terms}\\=2x+11x+9-4\\\mathrm{Add\:similar\:elements:}\:2x+11x=13x\\=13x+9-4\\\mathrm{Add/Subtract\:the\:numbers:}\:9-4=5\\=13x+5[/tex]

Suppose that in a certain sinkhole the ground dropped 69.6 ft in 24 hr. Find the unit rate representing the change in altitude per hour ..the unit rate representing the change in altitude is how many feet per hour

Answers

Answer:

2.9 per hour

Step-by-step explanation:

Divide 69.9 by 24

Solve for x. Show or explain your work.

Then, verify that your solution is correct.

-15 = 2x + 1

Answers

Answer:

-8 = x

Step-by-step explanation:

-15 = 2x + 1

-1            - 1            Subtract 1 from both sides

-16 = 2x                 Divide both sides by 2

-8 = x

To make sure this answer is correct, plug it into the equation to see if it works.

-15 = 2(-8) + 1       Multiply

-15 = -16 + 1          Add

-15 = -15

Data from the U.S. Department of Education indicates that 46% of business graduate students from private universities had student loans. Suppose you randomly survey a sample of graduate business students from private universities. Consider the sampling distribution (sample size n = 215) for the proportion of these students who have loans.What is the mean of this distribution?What is the standard deviation of this sampling distribution (i.e., the standard error)?

Answers

Answer:

For this case the mean is given by:

[tex] \mu = p =0.46[/tex]

And the standard deviation would be:

[tex] \sigma = \sqrt{\frac{0.46*(1-0.46)}{215}}= 0.0340[/tex]

Step-by-step explanation:

For this case we have the following info given :

[tex] n = 215[/tex] represent the sample size

[tex]p = 0.46[/tex] represent the proportion of business graduate students from private universities had student loans

For this case we want to find the distribution for the sample proportion and we know that this distribution is given by:

[tex] \hat p \sim N (p , \sqrt{\frac{p(1-p)}{n}}) [/tex]

And for this case the mean is given by:

[tex] \mu = p =0.46[/tex]

And the standard deviation would be:

[tex] \sigma = \sqrt{\frac{0.46*(1-0.46)}{215}}= 0.0340[/tex]

Answer:

The mean of this distribution is 0.46 and the standard deviation is 0.034.

Step-by-step explanation:

The Central Limit Theorem estabilishes that, for a normally distributed random variable X, with mean [tex]\mu[/tex] and standard deviation [tex]\sigma[/tex], the sampling distribution of the sample means with size n can be approximated to a normal distribution with mean [tex]\mu[/tex] and standard deviation [tex]s = \frac{\sigma}{\sqrt{n}}[/tex].

For a skewed variable, the Central Limit Theorem can also be applied, as long as n is at least 30.

For sampling distributions of samples of size n of a proportion p, the mean is [tex]\mu = p[/tex] and the standard deviation is [tex]s = \sqrt{\frac{p(1-p)}{n}}[/tex]

In this question:

[tex]n = 215, p = 0.46[/tex]

So

[tex]\mu = 0.46, s = \sqrt{\frac{0.46*0.54}{215}} = 0.0340[/tex]

The mean of this distribution is 0.46 and the standard deviation is 0.034.

To play the game you spin a spinner like the one shown you win if the arrow lands in one of the areas marked Win Lee played this game many times and recorded her results she won 11 times and lost 57 times use lee date to explain how to find the experimental probability of winning this game and to complete the explanation

Answers

Answer:

She played the game 11+57 = 68 times. She won 11 times. So the experimental probability of Lee winning is 11/68 = 0.1618 = 16.18%.

Step-by-step explanation:

To find the experimental probability of an outcome we divide:

The number of trials in which the desired outcome happened by the total number of trials.

In this question:

Experimental probability of winning this game.

She played the game 11+57 = 68 times. She won 11 times. So the experimental probability of Lee winning is 11/68 = 0.1618 = 16.18%.

Use the given information to find the P-value. Also, use a 0.05 significance level and state the conclusion about the null hypothesis (reject the null hypothesis or fail to reject the null hypothesis). With H1 : p ≠ 3/5, the test statistic is z = 0.78.
1) With H1: p ≠ 3/5, the test statistic is z = 0.78.
A) 0.4354; fail to reject the null hypothesis
B) 0.4354; reject the null hypothesis
C) 0.2177 fail to reject the null hypothesis
D) 0.2177; reject the null hypothesis
2) The test statistic in a left-tailed test is z = -1.83.
A) 0.0336; reject the null hypothesis
B) 0.0672; fail to reject the null hypothesis
C) 0.9664; fail to reject the null hypothesis
D) 0.0672; reject the null hypothesis
3) The test statistic in a right-tailed test is z = 0.52.
A) 0.0195; reject the null hypothesis
B) 0.3015; reject the null hypothesis
C) 0.3015; fail to reject the null hypothesis
D) 0.6030; fail to reject the null hypothesis

Answers

Answer:

1) With H1: p ≠ 3/5, the test statistic is z = 0.78

The p value for this case would be given by:

[tex] p_v = 2*P(z>0.78)=0.4354[/tex]

Best option:

A) 0.4354; fail to reject the null hypothesis

2) The test statistic in a left-tailed test is z = -1.83

The p value for this case would be given by:

[tex] p_v = P(z<-1.83)=0.0336[/tex]

Best option:

A) 0.0336; reject the null hypothesis

3) The test statistic in a right-tailed test is z = 0.52.

The p value for this case would be given by:

[tex] p_v = P(z>0.52)=0.3015[/tex]

Best option:

C) 0.3015; fail to reject the null hypothesis

Step-by-step explanation:

The significance level for all the cases is the same [tex]\alpha=0.05[/tex]

Part 1

With H1: p ≠ 3/5, the test statistic is z = 0.78

The p value for this case would be given by:

[tex] p_v = 2*P(z>0.78)=0.4354[/tex]

Best option:

A) 0.4354; fail to reject the null hypothesis

Part 2

The test statistic in a left-tailed test is z = -1.83

The p value for this case would be given by:

[tex] p_v = P(z<-1.83)=0.0336[/tex]

Best option:

A) 0.0336; reject the null hypothesis

Part 3

The test statistic in a right-tailed test is z = 0.52.

The p value for this case would be given by:

[tex] p_v = P(z>0.52)=0.3015[/tex]

Best option:

C) 0.3015; fail to reject the null hypothesis

Joe is a waiter of a local pizza parlor. He usually gets a tip from the tables he waits on. The bill for one table comes to $34. Write a formula that will help Joe determine how much of a tip he'll receive from that table.

Answers

Answer:

Step-by-step explanation:

I remember leaving a top of ten percent (15%)

lets convert 15% into a decimal

15%=0.15

Tip=0.15x, where x is the bill price

Tip=0.15*34

Tip=5.1

So he could get  5$ and 10 cents as a tip

The elephant is in danger of being completely wiped out. What sort of species is


an elephant?


a limiting species


O


an extinct species


a threatened species


an endangered species

Answers

Answer:

Endangered species

Step-by-step explanation:

An endangered species is a species that is at risk of extinction due to rapid decrease in their population .Their decrease in population might be due to loss of habitat and genetic variation. The loss of habitat might be due to natural factors like the climate change For example animals like dinosaurs during the cretaceous period that experience rapid change in the climate loss their habitat and went extinct.

Human activity can also influence loss of habitat. Development in housing , agriculture and industry can also threaten the habitats of native organisms. The loss of their habitat might cause sudden extinction of these organisms. They might find it hard to adapt to another environment or even procreate.  

Generally,  endangered species are species that are threatened by extinction. Elephants that are in danger of been wiped out is an endangered species.

Other Questions
A piggy bank contains only nickels, dimes, and quarters. There is a total of 105 coins and the total value of those coins is $10.75. Find a general solution of the number of each coin. List three possible specific solutions. Which social movement stemmed from the Seneca Falls Convention of 1848? a) the abolitionist movement b) the public education movement c) the womens rights movement d) the temperance movement which choice is equivalent to the product below when xx If 3x + 8 = 23, what is the value of x + 4 ? The thrust F of a screw propeller is known to depend upon the diameter d,speed of advance \nu ,fluid density p, revolution per second N, and the coefficient of viscosity of the fluid. Determine the dimensions of each of the variables in terms of L,M,T,and find an expression for F in terms of these quantities If g(x) = -4x + 5, find g(x+4). Summarize your contribution to Part III of the discussion. What was your analysis of the use of symbols in the novel, including where they appear, what they represent, and how they are used. How do they add to the story? Based on these symbols, what inferences can you make about the characters? How do these symbols relate to the plot? Which has the better value?? Plz help!!!!!!!!!!!A) One half-kilogram of bulk sunflower seeds costing 70 per 100 grams or B) A 500-milligram bag of prepackaged seeds costing $3.95 Combine A, B, C, D, E in one sentence A. She parker a car. B. It was her car. C. She parked illegally. D. She got a ticket. E. The ticket was for $50. What is the maximum value? Planters moved from the Old South to the Deep South to: On April 20, 2010, an explosion on the Deepwater Horizon Macondo oil well drilling platform released 4 million barrels (636,000 cubic meters) of oil into the Gulf of Mexico. It took scientists and engineers nearly three months and several different attempts to cap the well. The Macondo oil well was located 1524 m (5000 feet) below the surface of the water, about 80 km (50 miles) off the coast of Louisiana. The spill affected 2113 km (1300 miles) of shoreline along the coasts of five states. Most crude oil contains 8487% carbon by weight. Scientists have not yet fully assessed the environmental impact of the accident.Computer modeling can be used to help improve cleanup and response systems for the next major oil spill. Which of these uses demonstrates how computer modeling can be used to minimize the impact of another spill? aCreating a model of how the oil spread through the Gulf of Mexico to predict what areas need the greatest amount of protection. bCalculating the total oil that entered the Gulf of Mexico during the spill. cRecording the number and type of each dead organism collected during the spill. dFinding the average flow rate of the oil through the burst pipeline during the spill. two specific reasons that the serial comma should be used in your writing. PLEASE HELP IM BEGGING YOU! This is the name of the ancient Mississippian city that existed from the 600s-1400s and was the largest city in North America prior to European contract? please answer this (photo below) Arrange the following substances in order of increasing tendency to donate electronswith reasons: ketoglutarate, Oxaloacetate, O2, NADP+, Malate Dok = (6, 12) (3,6)The scale factor isA. 1/2B. 2C. 4 Which sentence below shows the correct use of a gerund?A.Conducting scientific experiments can be both fun and dangerous.B.Scientific experiments can be both fun and dangerous to conduct.C.The scientific experiments conducted are both fun and dangerous.D.Any scientific experiment conducted can be both fun and dangerous. Which is a characteristic of a studio apartment? a)typically has multiple bedroomsb)has multiple floorsc)has multiple units in the same buildingd)has one room for the kitchen, living room, and bedroom In young's double silt experiment if the distance between the silts is 0.5 and the distance between the silts and screen is 2 times.Then what will be the width of bands